Quantcast
  • Register
PhysicsOverflow is a next-generation academic platform for physicists and astronomers, including a community peer review system and a postgraduate-level discussion forum analogous to MathOverflow.

Welcome to PhysicsOverflow! PhysicsOverflow is an open platform for community peer review and graduate-level Physics discussion.

Please help promote PhysicsOverflow ads elsewhere if you like it.

News

PO is now at the Physics Department of Bielefeld University!

New printer friendly PO pages!

Migration to Bielefeld University was successful!

Please vote for this year's PhysicsOverflow ads!

Please do help out in categorising submissions. Submit a paper to PhysicsOverflow!

... see more

Tools for paper authors

Submit paper
Claim Paper Authorship

Tools for SE users

Search User
Reclaim SE Account
Request Account Merger
Nativise imported posts
Claim post (deleted users)
Import SE post

Users whose questions have been imported from Physics Stack Exchange, Theoretical Physics Stack Exchange, or any other Stack Exchange site are kindly requested to reclaim their account and not to register as a new user.

Public \(\beta\) tools

Report a bug with a feature
Request a new functionality
404 page design
Send feedback

Attributions

(propose a free ad)

Site Statistics

205 submissions , 163 unreviewed
5,047 questions , 2,200 unanswered
5,345 answers , 22,709 comments
1,470 users with positive rep
816 active unimported users
More ...

  Schrodinger equation from Klein-Gordon?

+ 5 like - 0 dislike
3024 views

One can view QM as a 1+0 dimensional QFT, fields are only depending on time and so are only called operators, and I know a way to derive Schrodinger's equation from Klein-Gordon's one.

Assuming a field $\Phi$ with a low energy $ E \approx m $ with $m$ the mass of the particle, by defining $\phi$ such as $\Phi(x,t) = e^{-imt}\phi(x,t)$ and developing the equation

$$(\partial^2 + m^2)\Phi~=~0$$

neglecting the $\partial_t^2 \phi$ then one finds the familiar Schodinger equation:

$$i\partial_t\phi~=~-\frac{\Delta}{2m}\phi.$$

Still, I am not fully satisfied about the transition field $\rightarrow$ wave function, even if we suppose that the number of particle is fixed, and the field know acts on a finite dimensional Hilbert Space (a subpart of the complete first Fock Space for a specific number of particles). Does someone has a reference to another proposition/argument for this derivation?

Edit: for reference, the previous calculations are taken from Zee's book, QFT in a Nutshell, first page in Chapter III.5.

This post imported from StackExchange Physics at 2014-04-01 16:22 (UCT), posted by SE-user toot
asked Jul 19, 2012 in Theoretical Physics by toot (445 points) [ no revision ]
retagged Apr 19, 2014 by dimension10
David Tong does have a good derivation/explanation for this in here: damtp.cam.ac.uk/user/tong/qft/two.pdf I have however had my own doubts about it, in case you have the same doubts, here are threads that answer them: physicsforums.com/showthread.php?t=709980 physics.stackexchange.com/questions/77290/…

This post imported from StackExchange Physics at 2014-04-01 16:22 (UCT), posted by SE-user guillefix

1 Answer

+ 6 like - 0 dislike

I think you are mixing up two different things. Namely:

  1. On the one hand, you can see QM as 0+1 (one temporal dimension) QFT, where the position operators (and their conjugate momenta) in the Heisenberg picture plays the role of the fields (and their conjugate momenta) in the QFT. You can check, for instance, that spatial rotational symmetry in the quantum mechanical theory is translated to an internal symmetry in the QFT.

  2. On the other hand, you can take the non-relativistic limit (by the way, ugly name because Galilean relativity is as relativistic as Special relativity) of the Klein-Gordon or Dirac theory to get the Schrödinger QFT, where $\phi$ (in your notation) is a quantum field instead of a wave function. There is a chapter in Srednicki's book where this issue is raised in a simple and nice way. There, you can also read about spin-stastistic and the wave function of multi-particle states. Let me add some equations that hopefully clarify (I'm using your notation and of course can be wrong factors, units, etc.):

The quantum field is: $$\phi \sim \int d^3p \, a_p e^{-i(p^2/(2m) \cdot t - p \cdot x)}$$

The Hamiltonian is:

$$H \sim i\int d^3x \left( \phi^{\dagger}\partial_t \phi - (1/2m)\partial _i \phi ^{\dagger} \partial ^i \phi \right) \, \sim \int d^3p \, p^2/(2m) \,a^{\dagger}_p a_p$$

The evolution of the quantum field is given by:

$$i\partial _t \phi \sim [\phi, H] \sim -\nabla ^2 \phi /(2m)$$

1-particle states are given by:

$$|1p> \sim \int d^3p \, \tilde f(t,p) \, a^{\dagger}_p \, |0> $$

(one can analogously define multi-particle states)

This state verifies the Schrödinger equation:

$$H \, |1p>=i\partial _t \, |1p>$$ iff

$$i\partial _t \, f(t,x) \sim -\nabla ^2 f(t,x) /(2m)$$

where $f(t,x)$ is the spatial Fourier transformed of $\tilde f (t,p)$.

$f(t,x)$ is a wave function, while $\phi (t, x)$ is a quantum field.

This is the free theory, one can add interaction in a similar way.

This post imported from StackExchange Physics at 2014-04-01 16:22 (UCT), posted by SE-user drake
answered Jul 19, 2012 by drake (885 points) [ no revision ]
Thanks for the update, but I am specifically under a limit operation that would lead me to a "first quantization scheme" from a "second quantization scheme", alias, is it enough from the fact that I recover a Schrodinger equation and can then construct a conserved probability current with a positive density component ($j^0$) to reinterpret the field as a wave function whose modulus square gives probabilities?

This post imported from StackExchange Physics at 2014-04-01 16:22 (UCT), posted by SE-user toot
I'm not sure what you are looking for. $f$ is a wave function that verifies the Schr. equation. The expectation value of $\phi$ is also a function that verifies the Schr. equation. So, as long as you can normalize them you get a quantum mechanical probabilistic interpretation. Have I answered your question?

This post imported from StackExchange Physics at 2014-04-01 16:22 (UCT), posted by SE-user drake
I think you did =) Thanks you.

This post imported from StackExchange Physics at 2014-04-01 16:22 (UCT), posted by SE-user toot
@drake: The expectation value of $\phi$ is not the correct way to extract wavefunction from field. The right way is to consider the state $\int \psi(x)\phi^\dagger(x)$ where $\psi$ is a number and $\phi^\dagger(x)$ is the nonrelativistic field.

This post imported from StackExchange Physics at 2014-04-01 16:22 (UCT), posted by SE-user Ron Maimon
@toot: The comment above is the correspondence between nonrelativistic fields and wavefunctions. If you smear a nonrelativistic creation field with a function, you produce a particle with wavefunction $\psi$.

This post imported from StackExchange Physics at 2014-04-01 16:22 (UCT), posted by SE-user Ron Maimon
Thanks. You are just changing my notation: your $\psi$ is my $f$. The expectation value is a solution of the equation, it is usually called the classical field.

This post imported from StackExchange Physics at 2014-04-01 16:22 (UCT), posted by SE-user drake

Your answer

Please use answers only to (at least partly) answer questions. To comment, discuss, or ask for clarification, leave a comment instead.
To mask links under text, please type your text, highlight it, and click the "link" button. You can then enter your link URL.
Please consult the FAQ for as to how to format your post.
This is the answer box; if you want to write a comment instead, please use the 'add comment' button.
Live preview (may slow down editor)   Preview
Your name to display (optional):
Privacy: Your email address will only be used for sending these notifications.
Anti-spam verification:
If you are a human please identify the position of the character covered by the symbol $\varnothing$ in the following word:
p$\hbar$ysicsOverflo$\varnothing$
Then drag the red bullet below over the corresponding character of our banner. When you drop it there, the bullet changes to green (on slow internet connections after a few seconds).
Please complete the anti-spam verification




user contributions licensed under cc by-sa 3.0 with attribution required

Your rights
...